Ableitung des Positionsoperators im QM

Zur Definition des Impulsoperators

P ^ = ich
In der Quantenmechanik können Sie dies, soweit ich weiß, ableiten, indem Sie entweder eine allgemeinere Definition des Impulses berücksichtigen, dh "kanonischen Impuls", der ein Operator ist, und diesen Operator dann auf Wellenfunktionen anwenden. Dies wird hier in diesem Wiki-Eintrag gezeigt . Wir können alternativ mit Übersetzungen beginnen und das Gesicht verwenden, dass der Momentum-Operator ein Generator von Übersetzungen ist, wie es hier in diesem Wiki-Eintrag getan wird .

Was mich interessiert, ist die grundlegendere Ableitung für den Positionsoperator:

X ^ = X .
Bisher habe ich überlegt, dass die Motivation für die Definition des Positionsoperators aus der Definition des Erwartungswerts stammt
X = D X X | ψ ( X ) | 2 = ψ | X | ψ
Wo | ψ ist normalisiert. Ist dies das volle Ausmaß der Motivation oder der Hauptpunkt, den es zu berücksichtigen gilt? Gibt es eine andere Motivation für den Positionsoperator oder wird dieser Motivation entnommen und durch Experimente als akzeptabel bestätigt?

Ich glaube, das ist eher eine Definition. Werte von Observablen sind durch die Eigenwerte ihres entsprechenden Operators gegeben. Sie würden also erwarten, dass der Positionsoperator Ihnen die Position gibt. Äquivalent, P natürlich arbeitet auf der P Staaten, um den Impuls zu geben, S z gibt die Spinprojektion usw.
Sie machen Verwirrung zwischen Operatoren und ihren Darstellungen auf (einigen) Basen . Die folgenden Antworten behandeln die Probleme richtig.
@GennaroTedesco: Da später möglicherweise zusätzliche Antworten unterschiedlicher Qualität hinzugefügt werden, möchten Sie möglicherweise keinen Kommentar posten, der "die Antworten unten" empfiehlt, ohne anzugeben, welche Antworten.
Siehe zB Stone-von-Neumann-Theorem auf Wikipedia . Wenn Ihnen diese Frage gefällt, können Sie auch diesen Phys.SE-Beitrag lesen.
@WillO Oh ja, klar, das war albern :D.
Siehe auch : physical.stackexchange.com/q/45248/2451 und darin enthaltene Links.

Antworten (3)

X ^ = X weil wir uns dafür entschieden haben, auf einer Basis von Eigenvektoren von zu arbeiten X ^ , dh Wellenfunktionen.

Wir haben einen Hilbertraum H das ist ein Vektorraum, auf dem wir jede gewünschte Basis wählen können. Wir entscheiden uns meistens dafür, mit einer Basis zu arbeiten, die den Positionsoperator diagonalisiert X ^ . Basiszustände erfüllen

X ^ | X = X | X
Sobald eine solche Grundlage gewählt ist, kann jeder Staat | Ψ In H kann darin erweitert werden, dh wir können schreiben
| Ψ = D X | X Ψ ( X )
Die Funktion Ψ ( X ) heißt Wellenfunktion. Wir können dies umkehren, um zu finden
Ψ ( X ) = X | Ψ

Nun, was ist die Bedeutung von ( X ^ Ψ ) ( X ) ? Per Definition ist es die Wellenfunktion des Zustands X ^ | Ψ , dh es ist X | X ^ | Ψ . Das ist dann sofort ersichtlich

X | X ^ | Ψ = X X | Ψ = X Ψ ( X )
So finden wir das ( X ^ Ψ ) ( X ) = X Ψ ( X ) . Aus diesem Grund schreiben wir der Einfachheit halber X ^ = X , aber man muss bedenken, dass dies nur in der Koordinatenbasis gilt.

Impulsoperator in Koordinatenbasis: Man könnte sich nun fragen, wie man den Ausdruck für den Impulsoperator herleiten kann P ^ in der Koordinatenbasis. Dies geschieht wie folgt.

Per Definition ( P ^ Ψ ) ( X ) = X | P ^ | Ψ . Dann,

( P ^ Ψ ) ( X ) = D P 2 π X | P P | P ^ | Ψ = D P 2 π X | P P | Ψ P = D X ' D P 2 π X | P P | X ' X ' | Ψ P
Als nächstes verwenden X | P = e ich P X , wir haben
( P ^ Ψ ) ( X ) = D X ' D P 2 π e ich P ( X X ' ) P Ψ ( X ' )
Wir schreiben jetzt
e ich P ( X X ' ) P = ich X e ich P ( X X ' )
Damit können wir explizit das Integral überführen P und wir finden
( P ^ Ψ ) ( X ) = ich X Ψ ( X )
Daher schreiben wir auf Koordinatenbasis
P ^ = ich X

PS - Selbstverständlich können wir die Basis frei wählen. Wir könnten zum Beispiel auf Impulsbasis arbeiten. Auf dieser Grundlage hätten wir

P ^ = P   , X ^ = ich P   .

Können wir das im Chat besprechen? Der Ausgangspunkt Ihrer Definition ist
X ^ | X = X | X .
Ist hier der Gedanke, dass wir mit dem Postulat von QM beginnen, das besagt, dass "jede Observable ein hermitischer Operator ist, der Eigenzustände hat, die eine Basis für den Hilbert-Raum sind". Es macht keinen Sinn, nicht von diesem Punkt auszugehen, da Sie dies bereits in den ersten beiden Gleichungen von Ihnen angenommen haben | X ist ein Eigenzustand eines Operators, den Sie noch nicht definiert hatten, und in der zweiten Gleichung haben Sie die Eigenvektoren angenommen | X sind komplett.
Was ist falsch daran, die Definition von Momentum (dargestellt in der Positionsbasis) abzuleiten, wie es in den von mir bereitgestellten Links (unter Verwendung des Übersetzungsoperators) getan wurde? Ist das Problem nur, dass es sich auf die Darstellung in der Positionsbasis beschränkt?
Ja zu Ihrem zweiten Kommentar. Wir haben bereits die hermiteschen Operatoren definiert X ^ Und P ^ durch ihre Kommutierungsbeziehung [ X ^ , P ^ ] = ich . Mit dieser Definition und dem von Ihnen formulierten Postulat von QM folgt alles. Ich bin im Chat verfügbar, wenn Sie es wünschen.
Okay, danke, wenn uns die Kommentare ausgehen, verschieben wir sie in den Chat. Was meinst du damit, wir haben die Operatoren bereits definiert X ^ Und P ^ durch ihre Kommutierungsrelation', erhalten Sie die Kommutierungsrelation nicht aus den Definitionen von X ^ Und P ^ ?
Nein. Wir beginnen mit dem klassischen System mit Koordinaten X und kanonisches Momentum P die eine Poisson-Klammer erfüllen { X , P } = 1 und einen Phasenraum M . Die Quantentheorie erhält man dann durch Definition eines Hilbertraums H wobei jeder Zustand einem Punkt entspricht M . X Und P werden zu Operatoren erhoben X ^ Und P ^ und Poisson-Klammern werden zu Kommutatoren erhoben. Das Ergebnis ist, dass wir jetzt ein Quantensystem haben, das durch einen Hilbert-Zustandsraum beschrieben wird H und zwei Operatoren X ^ Und P ^ das befriedigt [ X ^ , P ^ ] = ich .
Wir verwenden dann das Postulat, auf dem die Eigenvektoren hermitescher Operatoren basieren H und fahren Sie mit der Diskussion fort, die ich in meiner Antwort vorgestellt habe. Jede Basis kann gewählt werden und abhängig von der Basiswahl die Operatoren X ^ Und P ^ haben unterschiedliche Darstellungen. Die gebräuchlichste ist die Koordinatenbasis, mit der Sie vertraut sind. Eine weitere häufige ist die Energieeigenbasis.
Eine letzte Frage. Was ist die Motivation für die Definition X | P = e ich P X ? Woher kommt das?
Sie leitet sich ebenfalls aus der Kommutierungsrelation ab [ X ^ , P ^ ] = ich .
Okay und wissen wir, dass die Eigenzustände | X sind kontinuierlich im Gegensatz zu diskret, indem man einfach beobachtet, dass das Teilchen überall gefunden werden kann und in seiner Position nicht begrenzt ist?
Ja. Das ist richtig.

Beachten Sie, dass X ^ = X ist absolut nicht richtig. Die linke Seite ist der Operator, der auf die Hilbert-Zustände von Quantenzuständen wirkt. Die rechte Seite ist eine reelle Zahl.

Die RHS ist die Positionsdarstellung der LHS. Wie das OP richtig erkannt hat, ist dies in gewissem Maße eine Tautologie, da es sich um die definierende Darstellung des Operators über die Eigenwertgleichung handelt

X ^ | X = X | X .

Die a priori-Definition ist also diese mit der Vorstellung, dass die | X -Zustände sind lokalisierte Delta-Funktionen, dh

X ' | X = δ ( 3 ) ( X X ' ) .

Beachten Sie, dass die zweite obige Gleichung aus der ersten Gleichung (bis auf eine Konstante) unter der Annahme folgt, dass X ^ hermitesch ist und ihr Spektrum kontinuierlich ist.

Bisher war dies nur etwas Mathematik, wenn Sie darüber genauer lesen möchten, empfehle ich Quantenoptik im Phasenraum von W. Schleich .

Bisher war das nur Mathe. Das Wichtigste, was die Physik bestimmt, ist die Quantisierung, die darin besteht, die Beziehung aufzuerlegen:

[ X ^ , P ^ ] = ich

so ergibt sich auch die erste Relation in der Frage (dh der Impulsoperator in der Ortsdarstellung).

Tut X ' | X = δ ( X ' X ) folgt aus folgender Überlegung: X ' | X ^ | X = X | X ^ | X ' = X ' X | X ' = X ' X ' | X = X X ' | X . Dies gilt für alle X , X ' daher müssen wir das haben X ' | X = δ ( X j ) ?
@JohnDoe ja, du hast recht. die letzten beiden gleichgesetzt implizieren, dass es überall ungleich Null ist, außer wenn x=x'. und um vollständige Zustände zu erhalten, muss es sich um eine Delta-Funktion handeln (weshalb ich in der Frage "bis zu einer Konstante" gesagt habe; die gerade erwähnte behebt diese Konstante)
Nur um zu bestätigen, dass die Tatsache, dass die Operatoren, die Observablen entsprechen, hermtitisch sind, eher ein Postulat als etwas Abgeleitetes ist?
@JohnDoe ja, in der Tat. das ist eigentlich eines der Axiome der Quantenmechanik. Es ist lose motiviert durch die Tatsache, dass nicht-hermitische Operatoren komplexe Eigenwerte haben können, die für Observablen unphysikalisch wären.

In der QM haben Sie ein allgemeines Postulat, das erfordert, dass physikalische Observable (Impuls, Drehimpuls, Koordinaten) durch hermitische Operatoren dargestellt werden. Tatsächlich haben Sie im QM unterschiedliche Grundlagen sowie unterschiedliche Vorstellungen von Zeitentwicklung. Das sind Positions- und Koordinatenbasis sowie Schrödinger- und Heisenberg-Darstellungen. Wobei der Unterschied zwischen den letzten beiden darin besteht, dass im Heisenberg-Bild Operatoren zeitabhängig sind. Ihre Frage bezieht sich auf die Koordinatenbasis der Schrödinger-Darstellung, also konzentrieren wir uns darauf.

Bei der Koordinatendarstellung wird die Basis durch eine unendliche Menge von Vektoren gebildet | X > wofür folgendes gilt

X ^ | X >= X | X >

Jeder Zustandsvektor | ψ > kann in dieser Basis ausgedrückt werden als
| ψ >= D X | X >< X | ψ > ,
wobei wir den üblichen Ausdruck verwendet haben D X | X >< X | = ICH ^ . Der Positionsoperator war also einfach, was ist mit dem Impulsoperator? Hier brauchen Sie mehr Kalkül. In ähnlicher Weise gilt für die Impulsdarstellung, die wir haben
P ^ | P >= P | P >

Und
| ψ >= D P | P >< P | ψ > .
Um die beiden in Beziehung zu setzen, bemerken wir:
ψ ( X ) =< X | ψ >= D P < X | P >< P | ψ >= D P < X | P > ϕ ( P ) ,
was bedeutet, dass Sie über eine Fourier-Transformation von einer Basis zur anderen gehen. Wo die Fourier-Transformation gegeben ist durch
ψ ( X ) = 1 ( 2 π ) 2 3 D P e ich X P ϕ ( P )
In dieser Basis für den Impulsoperator P ^ wir haben
< X | P ^ | ψ >=< X | P ^ D P | P >< P | ψ >= 1 ( 2 π ) 2 3 D P P e ich X P ϕ ( P )
Seit
P e ich X P = ich e ich X P ,
wir haben
< X | P ^ | ψ >= ich < X | ψ > .
Oder einfach P ^ = ich und für den anderen Betreiber X ^ | X >= X | X > per Definition, weil wir in der Koordinatenbasis arbeiten.

Beides kannst du leicht überprüfen P ^ Und X ^ sind hermitesch und erfüllen die Heisenberg-Beziehung.

PS Sie können auch das Dirac-Bild (Wechselwirkung) haben, bei dem sowohl Zustände als auch Operatoren von der Zeit abhängen.